opposite rays form a?
line
ray
point
plane​

Answers

Answer 1

Answer:

ray is the answer for this

Answer 2

opposite rays form a line because they provide the two opposite directions in which the line extends infinitely.

Opposite rays form a what?

Opposite rays are two rays that have the same endpoint but extend in opposite directions. When these opposite rays are extended infinitely in both directions, they form a straight line. A line is a set of points that extends infinitely in both directions, and opposite rays provide the two distinct directions in which the line can be extended.

The concept of opposite rays is derived from the concept of a line. A line can be defined as a straight path that extends infinitely in both directions. Opposite rays are a pair of rays that share a common endpoint and extend infinitely in opposite directions along this line.

For example, consider a line segment AB. If we extend one side of the line segment from point A and the other side from point B, we obtain two opposite rays: one from point A to infinity and the other from point B to infinity. Together, these opposite rays form the line on which the line segment AB lies.

Learn more about lines at:

https://brainly.com/question/24607467

#SPJ6


Related Questions

h(x) = -x² + 3x + 10

Answers

Answer:

x = 5 or x = -2 or 3 - 2 x (derivative)

Step-by-step explanation:

Solve for x over the real numbers:

-x^2 + 3 x + 10 = 0

Multiply both sides by -1:

x^2 - 3 x - 10 = 0

x = (3 ± sqrt((-3)^2 - 4 (-10)))/2 = (3 ± sqrt(9 + 40))/2 = (3 ± sqrt(49))/2:

x = (3 + sqrt(49))/2 or x = (3 - sqrt(49))/2

sqrt(49) = sqrt(7^2) = 7:

x = (3 + 7)/2 or x = (3 - 7)/2

(3 + 7)/2 = 10/2 = 5:

x = 5 or x = (3 - 7)/2

(3 - 7)/2 = -4/2 = -2:

Answer: x = 5 or x = -2

____________________________________

Find the derivative of the following via implicit differentiation:

d/dx(H(x)) = d/dx(10 + 3 x - x^2)

Using the chain rule, d/dx(H(x)) = ( dH(u))/( du) ( du)/( dx), where u = x and d/( du)(H(u)) = H'(u):

(d/dx(x)) H'(x) = d/dx(10 + 3 x - x^2)

The derivative of x is 1:

1 H'(x) = d/dx(10 + 3 x - x^2)

Differentiate the sum term by term and factor out constants:

H'(x) = d/dx(10) + 3 (d/dx(x)) - d/dx(x^2)

The derivative of 10 is zero:

H'(x) = 3 (d/dx(x)) - d/dx(x^2) + 0

Simplify the expression:

H'(x) = 3 (d/dx(x)) - d/dx(x^2)

The derivative of x is 1:

H'(x) = -(d/dx(x^2)) + 1 3

Use the power rule, d/dx(x^n) = n x^(n - 1), where n = 2.

d/dx(x^2) = 2 x:

H'(x) = 3 - 2 x

Simplify the expression:

Answer:  = 3 - 2 x

Please Help
Function 1 is defined by the equation: p=r+7
Function 2 is defined by the table shown in the image below
Which function has a greater slope, function 1 or function 2?

Answers

Answer:

The slope of Function 2 (m=1.1) is greater than the slope of Function 1 (m=1).  

Step-by-step explanation:

First, note that p is essentially the y and that r is the x. Thus, to make this easier to see, convert p to y and r to x. Thus:

[tex]y=x+7[/tex]

From the above equation, we can determine that the slope is 1. Thus, the slope of Function 1 is 1.

To find the slope of the table, simply use the slope formula. Use any two points. I'm going to use the points (0,8) and (10,19). Let (0,8) be x₁ and y₁, and (10,19) be x₂ and y₂. Therefore:

[tex]m=\frac{y_2-y_1}{x_2-x_1}=\frac{19-8}{10-0}=11/10=1.1[/tex]

Thus, the slope of Function 2 is 1.1.

1.1 is greater than 1.

Thus, the slope of Function 2 is greater than the slope of Function 1.

Answer:

Function 2 has the greater slope

Step-by-step explanation:

Find the intervals on which f is increasing and the intervals on which it is decreasing. f(x)=-2cos^(2)x

Answers

Answer:

Increasing

0°≤x≤180°

Decreasing

180°≤x≤360°

What is the simplified form of the following expression? 2 StartRoot 18 EndRoot + 3 StartRoot 2 EndRoot + StartRoot 162 EndRoot

Answers

Answer: 2*√18 + 3*√2 + √162 = 18*√2

Step-by-step explanation:

I guess that the equation is:

2*√18 + 3*√2 + √162

And we want to simplify it.

first 18 = 9*2

then we can write:

2*√18 = 2*√(9*2) = 2*3*√2 = 6*√2

and 162/9 = 18

then we can write:

√162 = √(9*18) = √9*√18 = 3√18

now we can use the previous step: √18 = 3*√2

and:

√162 = 3*(3*√2) = 9*√2

now we can write our equation as:

6√2 + 3√2 + 9√2 = (6 + 3 + 9)√2 = 18*√2

And now we can not simplify it further more, so here we end.

Answer:

B. 18 sqrt 2

Step-by-step explanation:

This is the correct letter and answer on edge, if thats what youre using:)

what is the number if 4 is subtracted from the sum of one fourth of 5 times of 8 and 10

Answers

Answer:

Step-by-step explanation:

Lets, turn this into words and use order of operations, First, we look for multiplication and division.

the sum of one fourth of 5 times of 8 and 10 gets you 1/4(5*8) + 10 = 20

what is the number if 4 is subtracted from the sum

20 - 4 = 16

Question
The point (-2,r) lies on the graph of 2x + y = 7 in the xy-plane. What is the value of r?

Answers

Answer: r = 11

Step-by-step explanation:

We know that the point (-2, r) lies on the graph of:

2*x + y = 7.

Then, if we that point is on the graph of the equation, we can replace the values and we will have:

2*(-2) + r = 7

and now we solve this for r-

-4 + r = 7

r = 7 + 4 = 11

r = 11

Suppose that 200 students are randomly selected from a local college campus to investigate the use of cell phones in classrooms. When asked if they are allowed to use cell phones in at least one of their classes, 40% of students responded yes. Using these results, with 95% confidence, the margin of error is 0.068. How would the margin of error change if the sample size increased from 200 to 400 students?

Answers

Answer:

It would change to 0.04802

Step-by-step explanation:

from this question we have that n became 400

40% of 400

= 160

p* = 160/400

= 0.4

1 - p* =

= 1 - 0.4

= 0.6

at confidence level,

1 - 0.95

= 0.05

alpha/2 = 0.025

z= 1.96

margin of error. E

= 1.96 x √[(0.4 x 0.6)/400]

= 1.96 x 0.0245

= 0.04802

M.E = 0.04802

A recent survey of 1090 U.S. adults selected at random showed that 623 consider the occupation of firefighter to have very great prestige. Estimate the probability (to the nearest hundredth) that a U.S. adult selected at random thinks the occupation of firefighter has very great prestige.

Answers

Answer:

0.572

Step-by-step explanation:

From the question,

We have

n = 1090 of US adults

x = 623 selected from this population at random who consider the occupation to be one of great prestige

So we have that

The probability of X = x/n

= 623/1090

= 0.572

We conclude that 0.572 is the probability that a US adult selected at random thinks the occupation has great prestige.

A list of pulse rates is 70, 64, 80, 74,92. What is the median for this list?

Answers

Answer:

64 70 74 80 92

Answer = 74

Step-by-step explanation:

The median is when you have an order of numbers in ascending order (smallest to largest) then you find the middle number

Hope this helps :)

If anything is incorrect then please comment and I shall change the answer to the correct one

Median for the given data 70, 64, 80, 74,92 is equals to 74.

What is median?

"Median is defined as the central value of the given data after arranging them into ascending or descending order."

According to the question,

Given data for pulse rates = 70, 64, 80, 74,92

Arrange the data in ascending order we get,

64, 70 , 74, 80, 92

Number of pulse rate reading is 5 , which is an odd number.

Therefore, median is the central value.

Median for the given data = 74

Hence, median for the given data 70, 64, 80, 74,92 is equals to 74.

Learn more about median here

https://brainly.com/question/21396105

#SPJ2

] You are scheduled to receive $20,000 in two years. When you receive it, you will invest it for six more years at 8.4 percent per year. How much will you have in eight years?

Answers

Answer:

32449.3

Step-by-step explanation:

use the formula A = P(1+r / 100)^t

20000 × (1+ (8.4 / 100))^6

=32449.3

For some postive value of Z, the probability that a standardized normal variable is between 0 and Z is 0.3770. The value of Z is

Answers

Answer:

1.16

Step-by-step explanation:

Given that;

For some positive value of Z, the probability that a standardized normal variable is between 0 and Z is 0.3770.

This implies that:

P(0<Z<z) = 0.3770

P(Z < z)-P(Z < 0) = 0.3770

P(Z < z) = 0.3770 + P(Z < 0)

From the standard normal tables , P(Z < 0)  =0.5

P(Z < z) = 0.3770 + 0.5

P(Z < z) =  0.877

SO to determine the value of z for which it is equal to 0.877, we look at the

table of standard normal distribution and locate the probability value of 0.8770. we advance to the  left until the first column is reached, we see that the value was 1.1.  similarly, we did the same in the  upward direction until the top row is reached, the value was 0.06.  The intersection of the row and column values gives the area to the two tail of z.   (i.e 1.1 + 0.06 =1.16)

therefore, P(Z ≤ 1.16 ) = 0.877

Sean earned 20 points. Charles earned p more points than Sean. Choose the expression that shows how many points Charles earned.

Answers

The wording “Charles earned ‘p’ more points than Sean” tells us that Charles had the same number of points as Sean, plus whatever the amount that ‘p’ is.

Therefore, the expression to show how many points Charles earned would be:

p + 20

Answer:

the person above is correct if i did this correct

Step-by-step explanation:

Need Help
Please Show Work​

Answers

Answer:

1= 65 degrees

2=115 degrees

3=115 degrees

Step-by-step explanation: supplementary angles where 115 + x = 180 so go backwards by 180 - 115=65 to find corresponding angles. Angle 3 is also corresponding with the given angle of 115. Angle 2 is opposite the 115 so they have to be equal

Find the inverse of the following function.

Answers

Answer:

The inverse is 1/64 x^2 = y   x ≥ 0

Step-by-step explanation:

f(x) = 8 sqrt(x)

y = 8 sqrt(x)

Exchange x and y

x = 8 sqrt (y)

Solve for y

Divide each side by 8

1/8 x = sqrt(y)

Square each side

(1/8 x)^2 = (sqrt(y))^2

1/64 x^2 = y

The inverse is 1/64 x^2 = y   x ≥ 0

since x ≥0 in the original function

Answer:

[tex]\Huge \boxed{\mathrm{D}}[/tex]

Step-by-step explanation:

[tex]f(x)=8\sqrt{x}[/tex]

[tex]\sf Replace \ with \ y.[/tex]

[tex]y=8\sqrt{x}[/tex]

[tex]\sf Switch \ the \ variables.[/tex]

[tex]x= 8\sqrt{y}[/tex]

[tex]\sf Divide \ both \ sides \ of \ the \ equation \ by \ 8.[/tex]

[tex]\displaystyle \frac{x}{8} =\sqrt{y}[/tex]

[tex]\sf Square \ both \ sides \ of \ the \ equation.[/tex]

[tex]\displaystyle (\frac{x}{8} )^2 =y[/tex]

[tex]\displaystyle \frac{x^2 }{64} =y[/tex]

[tex]\displaystyle f^{-1}(x)=\frac{1}{64} x^2[/tex]

According to the local union president, the mean gross income of plumbers in the Salt Lake City area follows a normal distribution with a mean of $48,000 and a population standard deviation of $2,000. A recent investigative reporter for KYAK TV found, for a sample of 49 plumbers, the mean gross income was $47,600. At the 0.05 significance level, is it reasonable to conclude that the mean income is not equal to $47,600? Determine the p value. State the Null and Alternate hypothesis: State the test statistic: State the Decision Rule: Show the calculation: What is the interpretation of the sample data? Show the P value

Answers

Answer:

Step-by-step explanation:

Given that:

population mean [tex]\mu[/tex] = 47600

population standard deviation [tex]\sigma[/tex] = 2000

sample size n = 49

Sample mean [tex]\over\ x[/tex] = 48000

Level of significance = 0.05

The null and the alternative hypothesis can be computed as follows;

[tex]H_0 : \mu = 47600 \\ \\ H_1 : \mu \neq 47600[/tex]

Using the table of standard normal distribution, the value of z that corresponds to the two-tailed probability 0.05 is 1.96. Thus, we will reject the null hypothesis if the value of the test statistics is less than -1.96 or more than 1.96.

The test statistics can be calculated by using the formula:

[tex]z= \dfrac{\overline X - \mu }{\dfrac{\sigma}{ \sqrt{n}}}[/tex]

[tex]z= \dfrac{ 48000-47600 }{\dfrac{2000}{ \sqrt{49}}}[/tex]

[tex]z= \dfrac{400 }{\dfrac{2000}{ 7}}[/tex]

[tex]z= 1.4[/tex]

Conclusion:

Since 1.4 is lesser  than 1.96 , we fail to reject the null hypothesis and  that there is insufficient information to conclude that the   mean gross income is not equal to $47600

The P-value is being calculate as follows:

P -value = 2P(Z>1.4)

P -value =  2 (1 - P(Z< 1.4)

P-value = 2 ( 1 - 0.91924)

P -value = 2 (0.08076 )

P -value = 0.16152

if 2x-7 is 5 more than x+4, what is the value of 3x+5

Answers

Answer:

  53

Step-by-step explanation:

Let's start with the given relation:

  2x -7 = (x+4) +5

  x = 16 . . . . . . . . . add 7-x

  3x +5 = 3(16) +5 = 53 . . . . . multiply by 3 and add 5

The value of 3x+5 is 53.

What is the simplified form of x minus 5 over x squared minus 3x minus 10⋅ x plus 2 over x squared plus x minus 12 ? (6 points) Select one: a. 1 over the quantity x minus 3 times the quantity x plus 4 b. 1 over the quantity x minus 3 times the quantity x plus 2 c. 1 over the quantity x plus 4 times the quantity x minus 5 d. 1 over the quantity x plus 2 times the quantity x minus 5

Answers

Answer:

  [tex]\ \text{a. }\quad\dfrac{1}{(x-3)(x+4)}[/tex]

Step-by-step explanation:

Maybe you want the product ...

  [tex]\dfrac{x-5}{x^2-3x-10}\cdot\dfrac{x+2}{x^2+x-12}=\dfrac{x-5}{(x-5)(x+2)}\cdot\dfrac{x+2}{(x-3)(x+4)}\\\\=\boxed{\dfrac{1}{(x-3)(x+4)}}[/tex]

__

Numerator factors of (x-5) and (x+2) cancel those in the denominator.

If (x - 2) and (x + 1) are factors of
x + px? + qx + 1, what is the sum of p and q?

Answers

Answer:

p + q = -3

Step-by-step explanation:

First we need to take the original equation, and factor it to a form that's easier to get two binomial factors from (i.e., let's get a quadratic):

x^3 + px^2 + qx + 1

= x (x^2 + px + q) + 1

Now that we have factored out the x, we have a quadratic trinomial which we know can be broken down into two linear binomials.  The problem gives us two linear binomials, so let's take a look.

(x - 2) (x + 1) = (x^2 + px + q)

x^2 - 2x + x -2 = x^2 + px + q

Now let's solve.

x^2 - x - 2 = x^2 + px + q

-x - 2 = px + q

From here, we can easily see that p = -1 (the coefficient of x) and q = -2.

Hence, p + q = -1 + -2 = -3.

Cheers.

Answer two questions about Equations A and B:
A. 2x-1=5x
B. -1=3x

1) How can we get Equation B from Equation A?

Choose 1 answer:

Add/subtract the same quantity to/from both sides

Add/subtract a quantity to/from only one side

Rewrite one side (or both) by
combining like terms

Rewrite one side (or both) using the distributive property

NEXT QUESTION
based on the previous answer, are the equations equivalent? In other words, do they have the same solution?

A. Yes
B. No

Answers

Answer:

B: Add/subtract the same quantity to/from both sides

Next Question: Yes

Step-by-step explanation:

thats what the answer is dunno what else to tell you lol

Algebraic equations are mathematical equations that contain unknown variables.

To get Equation B from Equation A, we add/subtract the same quantity to/from both sides. Option A is the correct option. Equation A is equivalent to Equation B

Question 1: We are given equation A as:

2x - 1 = 5x .............Equation A

To get Equation B from A, we would subtract 2x from both sides of the equation.

2x - 2x - 1 = 5x - 2x

- 1 = 3x This is Equation B

Question 2: Based on the previous answer,

2x - 1 = 5x  is equal to  -1 = 3x.

Hence, both Equation A and Equation B are equivalent expressions.

Therefore,

To get Equation B from Equation A, we add/subtract the same quantity to/from both sides. Option A is the correct option.Equation A is equivalent to Equation B

To learn more, visit the link below:

https://brainly.com/question/22299566

Each leg of a 45°-45°-90° triangle measures 12 cm.
What is the length of the hypotenuse?
Z
х
45°
45°
O 6 cm
12 cm
12 cm
O 672 cm
O 12 cm
O 122 cm

Answers

Answer:

The legs are 12 cm each, so the hypotenuse is

√(144+144)=12√2

Step-by-step explanation:

Applying the Pythagorean Theorem, the length of the hypotenuse is:  12√2 cm.

The Pythagorean TheoremWhere, a and b are two legs of a right triangle, and c is the hypotenuse, the Pythagorean Theorem states that, c² = a² + b².

Given the two legs of the right triangle to be 12 cm

Therefore:

c² = 12² + 12².

c² = 288

c = √288

c = 12√2 cm

Therefore, applying the Pythagorean Theorem, the length of the hypotenuse is:  12√2 cm.

Learn more about, the Pythagorean Theorem on:

https://brainly.com/question/654982

49, 34, and 48 students are selected from the Sophomore, Junior, and Senior classes with 496, 348, and 481 students respectively. Group of answer choices

Answers

Answer:

Stratified Random sampling.

Step-by-step explanation:

As per the scenario, It is stratified random sampling as it divides students into strata which represent Sophomores, Juniors, and Seniors.

Simple random samples of the given sizes of the proportional to the size of the stratum which is to be taken from every stratum that is to be about 10 percent of students from every class that is selected here.

Hence, according to the given situation, the correct answer is a random stratified sampling.

In a triangle ABC two points D,E are taken on BC so that angle BAD=angle DAE=angleCAE. Determine AE if AB=5,BC=10 angle BAC=90. PLEASE HELP I NEED HELP WITHIN TEN MINS PLEASE

Answers

Answer:

AE = 7.5

Step-by-step explanation:

Since <BAC = [tex]90^{0}[/tex], then;

<BAD = <DAE = <CAE = [tex]30^{0}[/tex] (complementary angles)

From ΔABC, applying the Pythagoras theorem to determine the length of side AC;

[tex]/BC/^{2}[/tex] = [tex]/AC/^{2}[/tex] + [tex]/AB/^{2}[/tex]

[tex]/10/^{2}[/tex] = [tex]/AC/^{2}[/tex] + [tex]/5/^{2}[/tex]

100 = [tex]/AC/^{2}[/tex] + 25

[tex]/AC/^{2}[/tex] = 100 - 25

[tex]/AC/^{2}[/tex] = 75

AC = [tex]\sqrt{75}[/tex]

Applying trigonometric function to ΔCAE,

Cos [tex]30^{0}[/tex] = [tex]\frac{AE}{\sqrt{75} }[/tex]

AE = [tex]\sqrt{75}[/tex] × Cos [tex]30^{0}[/tex]

    = 7.5

Therefore, AE = 7.5

in need of assistance answers are greatly appreciated thank you for your time and effort

Answers

Answer:

x = (h+g)/-f

Step-by-step explanation:

-fx-g = h

Add g to each side

-fx-g+g = h+g

-fx = h+g

Divide each side by -f

-fx/-f = (h+g)/-f

x = (h+g)/-f

tan inverse 1/4 +tan inverse 2/7 = 1/2 cos inverse 3/5​

Answers

Answer:

The equation is always false

Step-by-step explanation:

arctan1/4+arctan2/7=1/2arccos3/5

0.24497866+0.27829965=1/2(0.92729521)

0.52327832                 =0.46364760

not equivalent and will never be.

Which of the following is NOT a requirement of testing a claim about two population means when 1 and 2 are unknown and not assumed to be​ equal? Choose the correct answer below. A. The two samples are dependent. B. Both samples are simple random samples. C. Either the two sample sizes are large ​(30 and ​30) or both samples come from populations having normal​ distributions, or both of these conditions are satisfied. D. The two samples are independent.

Answers

Answer:

b

Step-by-step explanation:

Jaclyn is one-fourth of a foot taller than John. John is 31/6 feet tall. How many feet tall is Jaclyn

Answers

Answer:

5 5/12

Step-by-step explanation:

31/6 feet + 1/4 foot

= 31/6 + 1/4

= [(31 * 4) / 6 * 4] + [(1 * 6) / 4 * 6]

=  [ 124/24 ] + [ 6/24 ]

= (124 + 6) / 24

= 130 / 24

= 5 10/24

= 5 5/12

Hope this helps!  Tell me if I'm wrong!

The price of a technology stock was $ 9.56 yesterday. Today, the price rose to $ 9.69 . Find the percentage increase. Round your answer to the nearest tenth of a percent.

Answers

Answer and Step-by-Step explanation:

% increase = 100 x [(new price) - (original price)] / (original price)] = 100 (9.67 - 9.56) / 9.56

% increase ≅ 1.2% (to the nearest tenth)

A low-noise transistor for use in computing products is being developed. It is claimed that the mean noise level will be below the 2.5-dB level of products currently in use. It is believed that the noise level is approximately normal with a standard deviation of .8. find 95% CI

Answers

Answer:

The 95% CI is   [tex]2.108 < \mu < 2.892[/tex]

Step-by-step explanation:

From the question we are told that

   The  population mean [tex]\mu = 2.5[/tex]

    The standard deviation is  [tex]\sigma = 0.8[/tex]

Given that the confidence level is  95% then the level of confidence is mathematically evaluated as

          [tex]\alpha = 100 - 95[/tex]

   =>  [tex]\alpha = 5\%[/tex]

  =>    [tex]\alpha = 0.05[/tex]

Next we obtain the critical value of  [tex]\frac{\alpha }{2}[/tex] from the normal distribution table, the values is  [tex]Z_{\frac{\alpha }{2} } = 1.96[/tex]

Generally the margin of error is mathematically evaluated as

          [tex]E = Z_{\frac{\alpha }{2} } * \frac{\sigma}{\sqrt{n} }[/tex]

here we would assume that the sample size is  n =  16 since the person that posted the question did not include the sample size

  So    

               [tex]E = 1.96* \frac{0.8}{\sqrt{16} }[/tex]

               [tex]E = 0.392[/tex]

The  95% CI is mathematically represented as

              [tex]\= x -E < \mu < \= x +E[/tex]

substituting values

              [tex]2.5 - 0.392 < \mu < 2.5 + 0.392[/tex]

substituting values

              [tex]2.108 < \mu < 2.892[/tex]

       

Simplify . 7+ the square root of 6(3+4)-2+9-3*2^2 The solution is

Answers

Answer:

7+sqrt(37)

Step-by-step explanation:

7+sqrt(6*(3+4)-2+9-3*2^2)=7+sqrt(6*7+7-3*4)=7+sqrt(42+7-12)=7+sqrt(37)

Solve for x: −3x + 3 −1 b. x −3

Answers

Answer:

2/3

Step-by-step explanation:

Your −3x + 3 −1 is not an equation and thus has no solution.

If, on the other hand, you meant

−3x + 3 = 1

then -3x = -2, and  x = 2/3

Other Questions
If the rods with diameters and lengths listed below are made of the same material, which will undergo the largest percentage length change given the same applied force along its length?a. d, 3L b. 3d, L c. 2d, 2L d. 4d, L Please help with this Please help me answer this question. -15 - g/3 = -5.What is g? Harwell Company manufactures automobile tires. On July 15, 2018, the company sold 1,300 tires to the Nixon Car Company for $50 each. The terms of the sale were 3/10, n/30. Harwell uses the gross method of accounting for cash discounts. Required: 1. Prepare the journal entries to record the sale on July 15 (ignore cost of goods) and collection on July 23, 2018. 2. Prepare the journal entries to record the sale on July 15 (ignore cost of goods) and collection on August 15, 2018 what is the central idea of the text? Harry and Sally formed the Evergreen partnership by contributing the following assets in exchange for a 50 percent capital and profits interest in the partnership. Basis Fair Market Value Harry: Cash $30,000 $30,000 Land $100,000 $120,000 Totals $130,000 $150,000 Sally: Equipment used in business $200,000 $150,000 Totals $200,000 $150,000 a. How much gain or loss will Harry recognize on the contribution? b. How much gain or loss will Sally recognize on the contribution? c. Should Sally consider selling the property to the partnership rather than contributing it? A. YesB. No Choose the correct answer.Su vuelo sali con una demora; por eso, ella va a llegara bordoa tiempoa vecestarde Will Give Brainliest, answer ASAP x= y= z= What is the reason for using short sentences at the beginning of this article?O The author wanted to become more famous.O The author wants to describe things clearly.O The author wants to analyze important details.O The author wants to get the readers' attention. Help the question is there A blimp is 1100 meters high in the air and measures the angles of depression to two stadiums to the west of the blimp. If those measurements are 75.2 and 17.9, how far apart are the two stadiums? According to the author Rudyard Kipling, what was the "white man's burden" acall to do?O A. Westernize colonies in Africa and AsiaB. End slavery in America and EuropeC. Invent new types of technologyO D. Discover cures for diseases Select the correct answer from each drop-down menu.The function f is given by the table of values as shown below.x12345f(x)13193791253Use the given table to complete the statements.The parent function of the function represented in the table is .If function f was translated down 4 units, the -values would be .A point in the table for the transformed function would be . The clear water was like a Carlos and Deborah are farmers. Each one owns a 12-acre plot of land. The following table shows the amount of rye and corn each farmer can produce per year on a given acre. Each farmer chooses whether to devote all acres to producing rye or corn or to produce rye on some of the land and corn on the rest.Rye Corn(Bushels per acre) (Bushels per acre)Carlos 18 6Deborah 28 7___________ has an absolute advantage in the production of rye, and _________ has an absolute advantage in the production of corn. Carlos's opportunity cost of producing 1 bushel of corn is___________ bushels of rye, whereas Deborah's opportunity cost of producing 1 bushel of corn is ___________ bushels of rye. Because Carlos has a ___________ opportunity cost of producing corn than Deborah,____________ has a comparative advantage in the production of corn, and____________ has a comparative advantage in the production of rye. PLEASE HELP!!!! i need to compare and contrast between Mohandas Gandhi and augusto pinochet!! needs 500-600 words or just try and make it long please!!! Solve for x. 3x-91>-87 AND 17x-16>18 Latanya buys 5 yard of blue fabric and 8 yards of green fabric. the blue fabric cost $2 dollars more than the green fabric.she pays a total of $ 62. what would be the combined cost of 1 yard of blue fabric and one yard of green fabric? Which of the following best explains the potential health problems associated with endocrine disruptors? A) Endocrine disruptors are the only available pesticides,hence they are used widely. B) Endocrine disruptors mimic hormones that regulate critical biological processes. C) Endocrine disruptors do not biodegrade, remaining toxic years after their release. D) There are no major health problems associated with exposure to endocrine disruptors. E) Endocrine disruptors encompass a wide array of toxic pesticides. Evidence without_______increases its reliability.abiasbquestionrepetitiondreplication